User Avatar
quicktjc771
Joined
Apr 2025
Subscription
Free
User Avatar
quicktjc771
Wednesday, Dec 28 2022

Hello, I'd personally wait for the next cycle. I would have my personal statement and everything in order and ready to go. Come September I would start applying asap. I took a full year off and come 2023 I will score high and be ready by September. If you get the score that your happy with in February then I would apply asap shortly after. If not, then wait till September. By then, hopefully you would haven taken the LSAT and gotten a score your happy with. Patience, everything will fall into place.

User Avatar
quicktjc771
Saturday, Jan 28 2023

Hi, thank you for taking the time out to reply. I think I was way over thinking this big time and it didn't help that I was exhausted. I was trying to show that if N-m->/A, then the conclusion is invalid because as you stated there would be a some relationship in the conclusion rather then a most. Now in regards to the second list, I meant to put / next to the A. Bottom line is that N-m->/A doesn't allow the conclusion to be true and N—>/A does. Is this correct?

Someone tell me if I'm correct in this logic. This is for question #2. Here is the link.

https://classic.7sage.com/lesson/quiz-on-finding-sufficient-assumptions-with-intersection-statements-2-answers/?ss_completed_lesson=11895

G –m→ N

G –m→ /A

G-m->N-m->/A = I’m not able to draw it out like how JY does so this is what I put however, on paper I draw it out like JY does.

N—>/A = Answer

So in regards to N-m->/A not being the answer, let me explain.

/A

/A

GN

GN

GN/A

GN/A

G/A

G/A

N/A

N/A

Now the reason N-m->/A is incorrect is because this answer would effect the conclusion. Say you had more A’s and more N’s then it wouldn’t be the case that G-m->/A is the answer because look, I have more A’s and N’s that are not G’s so how could G –m→ /A? This would make the conclusion invalid. It has to be certain that most G’s are /A and by getting ride of N-m->/A statement were getting ride of the extra two N’s that carry A with it as well as the other extra two /A’s. If N-m->/A then how does G-m->/A work? Out of the 6 N’s, two are A which pulls from the conclusion by saying, “No, G-m->/A invalid because we have two extra A’s therefore out of all the A’s which is 8 (100%) only 4 G’s are A’s and it needs to be 5 to count for most. The N-m->/A is satisfied but doesn’t allow the conclusion to be valid so the sufficient assumption that N—>/A makes it 100% true that yes G –m→ /A.

GN

GN

GN/A

GN/A

G/A

G/A

PrepTests ·
PT102.S4.Q26
User Avatar
quicktjc771
Wednesday, Feb 22 2023

Wow, I shouldn't have missed this LOL! Weak sauce.

User Avatar
quicktjc771
Thursday, Mar 21 2024

Well when do you plan on taking the LSAT?

User Avatar
quicktjc771
Wednesday, Nov 20 2024

I would love to join as well.

User Avatar
quicktjc771
Wednesday, Jul 17 2024

Loyola Law School here we come!

PrepTests ·
PT105.S2.Q6
User Avatar
quicktjc771
Wednesday, Nov 16 2022

This is one hell of a question. The key to understanding this question is to understand that that the nesting boxes aren't well hidden because the boxes are crowed. Now if the boxes are well hidden then parasitic behavior is rarer. It's safe to infer that some ducks lay there eggs in boxes because they aren't well hidden.

Hope this helps! This took me awhile to understand and then suddenly, the logic clicked.

User Avatar
quicktjc771
Monday, Jun 16

Im innnnnnnn :)

PrepTests ·
PT106.S1.Q14
User Avatar
quicktjc771
Wednesday, Feb 15 2023

I got this question correct however, what messed me up was the sufficient condition "without". Can someone please do the conditional logic for the "without"?

#help

PrepTests ·
PT104.S4.Q17
User Avatar
quicktjc771
Wednesday, Feb 15 2023

Wow, I feel that was an easy question I shouldn't have missed. I picked E however, had I read carefully I would of read that E is talking about other non profit groups as well. Answer choice A is the only answer choice that must be true based on the stimulus.

User Avatar
quicktjc771
Tuesday, Apr 11 2023

I'm in LOL PEEP THE NAME!

PrepTests ·
PT102.S4.Q7
User Avatar
quicktjc771
Friday, Feb 10 2023

Ok so this is a pretty confusing question because I had a hard time understanding the passage however; let me explain why A is the correct answer choice for myself and others. Basically, if for instance fisherman could catch 10,000 fish a day before the law but after the law was imposed, fisherman can only catch 1,000 fish a day then the demand for these fish will increase. Prices are going to increase because many people want these fish. I hope this helps someone.

User Avatar
quicktjc771
Wednesday, May 10 2023

@ Hello, if possible may I receive it as well? Thank you so much!

PrepTests ·
PT102.S2.Q4
User Avatar
quicktjc771
Tuesday, Jan 10 2023

Wow ok so I chose E and then C on blind review. Answer E weakens the argument because if MANY criminals don't have access to photocopiers then why should the special ink even be adopted? Now answer choice C messed me up because I made a big assumption. I thought well if microprinting paper has fewer steps then say special ink then yes we should adopt special ink because it's a process that takes longer and criminals wont be able to do that process. Lmao such a terrible assumption. Answer choice A definitely strengthens the argument because if cost is being considered then it makes sense that one would want special ink currency because THIS CURRENCLY CAN BE DETECTED EASILY. I feel that this is a shitty answer choice but rather, it's the only answer choice that strengthens the argument.

PrepTests ·
PT103.S3.Q21
User Avatar
quicktjc771
Friday, Feb 10 2023

Dude fuck this question. SMH I hate you LSAT writers.

User Avatar
quicktjc771
Friday, Aug 09 2024

Stay positive and be persistent. We got this! Don't give up. Make a schedule and stick to it.

PrepTests ·
PT102.S2.Q25
User Avatar
quicktjc771
Thursday, Feb 09 2023

Ok so I'm overthinking this and I'm confused. I picked answer choice C and I see why it's wrong but in regards to answer choice D, how is this the answer? Jack didn't pick neither George nor his mom. If jack picked his mom, then she would benefit more than George because there is more people; yet he didn't. What am I missing here?

#help

PrepTests ·
PT102.S4.Q18
User Avatar
quicktjc771
Thursday, Feb 09 2023

Dumbass question smh! I hate you so much LSAC.

User Avatar

Saturday, Oct 08 2022

quicktjc771

Coffee or Tea?

Hello everyone! Ok so I enjoy drinking coffee however I noticed I crash hard when I'm coming down later in the day. I've tired drinking matcha tea however it's not as strong as coffee and I'm not able to focus as much. The problems I'm having with coffee is I believe it affects my sleep. I drink literally one cup at 9am and for some reason my mind is constantly racing. at night. In other words I'm not getting REM sleep. Does anyone else have problems with caffeine? I'm considering just quitting coffee cold turkey and only drinking matcha from now on. WHAT DO I DO?!?!?!?!?

PrepTests ·
PT113.S3.Q16
User Avatar
quicktjc771
Friday, Jan 06 2023

This is one hell of an argument. I understand (sort of), why answer E is the answer however this answer choice seems like it destroys the premise which is wont we don't want. We need to destroy the support. What am I missing here? I don't understand exactly what E is doing?

#help (Added by Admin

PrepTests ·
PT107.S1.Q19
User Avatar
quicktjc771
Thursday, Apr 06 2023

I picked answer choice C however, I realize why it's wrong. The trick to getting this answer correct was realizing that the manager was aware of the delay and the word foreseeable correlates with foreseeable. Also, now that I think about it, POE helps with this question because the other answer choices are plain wrong for a variety of reasons.

PrepTests ·
PT117.S2.Q17
User Avatar
quicktjc771
Wednesday, Apr 05 2023

Wow, I picked answer choice B and it's rather obvious why this wouldn't be the answer choice.

User Avatar
quicktjc771
Wednesday, Apr 05 2023

Hello, I'm sure you've gotten some pretty good responses however, I would like to chime in. I'm in the same exact boat as you. Here's the thing. Your studying way to much and from the sound of it, it sounds like your burning out and your going overboard. Take a step back and reevaluate yourself. I wouldn't study more than 4 hours / 5 days a week. Sure you can do more however, remember these words: "quality or quantity". Make a schedule and stick to do. Be sure to get some exercise in as well. I'm telling you, this takes time! If it makes you feel better, I took the Princeton review and fucking bombed meaning I did horrible on my first LSAT and now I'm on 7sage and doing it all over again. Stop being hard on yourself and learn to take breaks. We're not gods, but we're humans and we can only do so much each day before our brain say's "ok, no more. I need a break!". Feel free to message me if you'd like any advice.

PrepTests ·
PT102.S2.Q15
User Avatar
quicktjc771
Thursday, Jan 05 2023

Ok so I'm confused as hell. I understand why answer choice D is correct however, the conclusion mentions that a thermometer could be accurate if PLUS 1 or Minus 1. Answer choice D says the temperature is different if PLUS 2. This answer choice seems like it would strengthen rather than weaken because PLUS 1 could be way more than 2 which aligns with answer choice D. What am I missing here? #help #help

PrepTests ·
PT106.S3.Q15
User Avatar
quicktjc771
Thursday, Jan 05 2023

Wow! Ok so what messed me up is I didn't realize that intellectual is similar to cognitive. I picked answer choice E and this answer doesn't do shit. I need to assume that serious athletes are aware of their cognitive abilities however, that's an extreme assumption I need to make for answer choice E. This is a well written question that confused the hell out of me.

Confirm action

Are you sure?